Proof Limit of (4n^(3)+3n)/(n^(3)-6): Simple Solution

  • Thread starter Thread starter torquerotates
  • Start date Start date
  • Tags Tags
    Limit Proof
Click For Summary
The discussion centers on proving the limit of the expression (4n^(3)+3n)/(n^(3)-6) as n approaches infinity. The book's solution uses an upper bound to show that the limit approaches 4, while the user proposes an alternative approach using a lower bound. They argue that both methods could be valid, but express a preference for their simpler solution. The conversation highlights the importance of establishing bounds for sequences and the implications of monotonicity in convergence. Ultimately, the user seeks confirmation of the correctness of their reasoning.
torquerotates
Messages
207
Reaction score
0
Now this is an example that I don't really understand. It says prove that the limit of (4n^(3)+3n)/(n^(3)-6)

Homework Equations



Lim{a}=L

For any epsilion>0 there is a N>0 such that n>N=> |{a}-L|<epsilion

The Attempt at a Solution



So here's the book's solution.

|(4n^(3)+3n)/(n^(3)-6) - 4|=|(3n+4)/(n^(3)-6)|<epslion

but if n>or=2, (3n+4)/(n^(3)-6)<epslion

they found an upper bound for the sequence.

(3n+24)<or =30n

& (n^(3)-6)>or =(1/2)n^(3)
So, (3n+4)/(n^(3)-6)<or= (30n)/( (1/2)n^(3))<epsion

=> 60/n^(2)<epslion
=> (60/epslion)^(-1/2)<n

so this implies that we make N=max{2, (60/epslion)^(-1/2)} Now here's my solution,

I never found a reason why they used an upper bound. Wouldn't a lower bound work just as well? (3n+4)/(n^(3)-6)<epslion

n/(n^(2)) <(3n+4)/(n^(3)-6)<epslion. Now I choose the denominator,n^2, arbitaritly just to make just sure that the denominator becomes smaller.

So solving, 1/epslion<n. This implies that we make N=max{2, 1/epslion}

So far, I'm convinced that my solution is the most simple. But is it correct?
 
Physics news on Phys.org
\epsilon is "epsilon" and you can simply write "<or = " as "<=".

Notice that your sequence is monotonically increasing. Let S= \left\{ a_{n}|n \in N \right\} so \epsilon &gt; 0, you need to find an index N such that

|a_{n}- l| &lt; \epsilon

which is the same as

a_{n} \leq l &lt; l + \epsilon

Notice that l is an upper bound for the set S, so

\implies l - \epsilon &lt; a_{n} &lt; l+ \epsilon
So, l is the least upper bond or supS . Thus, there is an index N such that l - \epsilon &lt; a_{N}; but, the sequence is mononotically increasing so

l - \epsilon &lt; a_{N} \leq a_{n}

In Short: Since your sequence is monotonically increasing, then it converges if and only if it is bounded above.
 
Question: A clock's minute hand has length 4 and its hour hand has length 3. What is the distance between the tips at the moment when it is increasing most rapidly?(Putnam Exam Question) Answer: Making assumption that both the hands moves at constant angular velocities, the answer is ## \sqrt{7} .## But don't you think this assumption is somewhat doubtful and wrong?

Similar threads

  • · Replies 2 ·
Replies
2
Views
2K
  • · Replies 9 ·
Replies
9
Views
3K
Replies
29
Views
5K
  • · Replies 12 ·
Replies
12
Views
2K
Replies
14
Views
2K
  • · Replies 2 ·
Replies
2
Views
3K
  • · Replies 4 ·
Replies
4
Views
8K
Replies
5
Views
1K
Replies
7
Views
4K
  • · Replies 4 ·
Replies
4
Views
1K